Valeur propre d'une edo

Bonjour
on considère le problème aux limites $$

\begin{cases}
y''+ \lambda y &=~0\\
y(0)-2 y(2 \pi)&=~0\\
y'(0)-y'(2\pi)&=~0
\end{cases}

$$ On dit que $\lambda$ est une valeur propre du problème au limite si et seulement si la solution associée n'est pas triviale.
La question est : comment montrer qu'un nombre complexe ne peut pas être une valeur propre de ce problème aux limites ?
Cordialement.

Réponses

  • Bonjour,

    Tu ne peux pas.

    Mais si tu écrit $\lambda = a+i b$ avec $a,b$ réels tu dois pouvoir démontrer des trucs sur $b$.
  • Ton problème est:
    1. Montrer qu'un nombre complexe $a +ib \in \C$ en particulier n'est pas valeur propre, ou
    2. Montrer qu'il n'existe pas de valeurs propres ayant une partie imaginaire non nulle ?
  • Mon problème est de montrer qu'il n y a pas de valeur propre ayant une partie imaginaire non nulle.

    Cordialement
  • up! Quelqu'un peux m'aider?

    Cordialement
  • Bonjour,

    Vérifie ceci :
    (A faire) : Pour tout $\lambda$ complexe, il existe $\displaystyle a,b$ réels tels que $\displaystyle \lambda = -r^2$ avec $\displaystyle r=a+ib.$

    Si $\displaystyle r=0$, on a les solutions triviales. Donc on traite les cas $\displaystyle r \neq 0.$

    On cherche les solutions sous la forme $\displaystyle y(x) e^{rx}$ : $\displaystyle r^2+\lambda = 0$.

    Les solutions sont (deux solutions distinctes pour une équation différentielle linéaire d'ordre deux) donc $\displaystyle y(x) = A e^{r x} + B e^{-r x} $ avec $\displaystyle A,B$ deux complexes.

    On note $\displaystyle z = e^{2 \pi r} \neq 0.$

    On écrit les conditions aux bords : $\displaystyle A+B = 2Az+2 B/z$ et $\displaystyle A-B = Az-B/z.$

    On résout ce système proprement (à faire). On trouve :
    - soit $\displaystyle z \neq 1$, et alors $A=B=0$ : c'est la solution triviale.
    - soit $\displaystyle z=1$, et alors $\displaystyle A+B=0.$ Et donc la condition aux bords donne $\displaystyle 2Ar \sinh(2 \pi r) = 0.$

    On résout cette équation proprement (à faire). On trouve : $\displaystyle r = i {m \over 2}, m \in \Z.$ Et donc $\displaystyle \lambda = -r^2=+({m \over 2})^2 \in \R$ pour tout $\displaystyle m\in \Z.$

    On conclut.
  • Bonjour
    Multiplier chaque membre par $\bar{y}$, faire une I.P.P et utiliser conditions de bord.
    Prendre alors la partie imaginaire pour voir que $\Im(\lambda)=0$ ,
     
  • Bonjour
    pour l'idée de bd2017 quand on multiplie les deux membres de l'équation par $\bar{y}$, ça donne:
    $$
    y''\bar{y}+\lambda y \bar{y}=0
    $$
    On pose $\lambda = a+i b$ et $y= u + i v$. Ce qui donne
    $$
    u''u+i(uv''-u''v)+v''v+(a+ib)(u^2+v^2)=0.
    $$
    Après comment on fait la suite exactement? L'idée n'est pas claire.

    Merci d'avance.
  • Intègre sur $[0,2\pi]$.
  • Oui ça je le sais mais l'intégration nous donne plein de terme:
    $$
    (a+ib) \displaystyle\int_0^{2\pi} (u^2(x)+v^2(x)) dx + \displaystyle\int_0^{2\pi} u''(x)u(x) dx + \displaystyle\int_0^{2\pi} v''(x) v(x) dx + i\displaystyle\int_0^{2\pi} (uv''-u''v)dx=0.
    $$
    Donc chacune des parties réelles et imaginaire est nulle. Ce qui nous intéresse est la partie imaginaire, donc on a
    $$
    b\displaystyle\int_0^{2\pi} (u^2+v^2) dx = \displaystyle\int_0^{2\pi} u'' v dx - \displaystyle\int_0^{2\pi} uv'' dx.
    $$
    Par l'ipp on trouve que
    $$
    \displaystyle\int_0^{2\pi} u'' v dx -\displaystyle\int_0^{2\pi} u v'' dx = v(2\pi)u'(2\pi)-v(0)u'(0)-u(2\pi)v'(2\pi)+u(0)v'(0)
    $$
    et par les conditions aux limites, c'est égale à
    $$
    -v(2\pi)u'(2\pi)+u(2\pi) v'(2\pi).
    $$
    Ceci ne donne rien de bon, ça devrait être zéro. Où est le problème?

    Cordialement
  • Prends la partie imaginaire et fais des intégrations par parties.
  • Oui il y a eu un doublon. J'ai modifié mon message avant de voir le votre et ça donne ce que j'ai écris. Ce n'est pas bon.

    $$
    (a+ib) \displaystyle\int_0^{2\pi} (u^2(x)+v^2(x)) dx + \displaystyle\int_0^{2\pi} u''(x)u(x) dx + \displaystyle\int_0^{2\pi} v''(x) v(x) dx + i\displaystyle\int_0^{2\pi} (uv''-u''v)dx=0.
    $$
    Donc chacune des parties réelles et imaginaire est nulle. Ce qui nous intéresse est la partie imaginaire, donc on a
    $$
    b\displaystyle\int_0^{2\pi} (u^2+v^2) dx = \displaystyle\int_0^{2\pi} u'' v dx - \displaystyle\int_0^{2\pi} uv'' dx.
    $$
    Par l'ipp on trouve que
    $$
    \displaystyle\int_0^{2\pi} u'' v dx -\displaystyle\int_0^{2\pi} u v'' dx = v(2\pi)u'(2\pi)-v(0)u'(0)-u(2\pi)v'(2\pi)+u(0)v'(0)
    $$
    et par les conditions aux limites, c'est égale à
    $$
    -v(2\pi)u'(2\pi)+u(2\pi) v'(2\pi).
    $$
    Ceci ne donne rien de bon, ça devrait être zéro. Où est le problème?
    Je pense qu'il y a une erreur dans les conditions aux limites données dans l'exo. Non?
  • Bonjour,

    Reste avec les quantités $y,\bar{y}$ et leurs dérivées et $\lambda.$

    Tu sais suivre des indications ?

    Tu multiplies l’équation différentielle par $\bar{y}$ et tu intègres entre $0$ et $2\pi.$ pour le terme en $y”$ tu intègres par parties.
  • Oui j'ai suivi les instruction: on a bien $$

    \displaystyle\int_0^{2\pi} y'' \bar{y} dx+ \lambda \displaystyle\int_0^{2\pi} y \bar{y} dx,

    $$ mais si on ne remplace pas $y$ par $u+iv$ alors comment on peut calculer directement $\displaystyle\int_0^{2\pi} y'' \bar{y} dx$ ?!
    Merci d'avance.
  • Bonjour,

    L’indication dit ‘reste avec $y$ son conjugué et ses dérivées’. Pas de $u$ ni de $v.$

    L’indication dit ‘fait une intégration par parties pour le terme en $y”.$’

    On ne demande pas de calculer l’intégrale mais plutôt de l’exprimer différemment pour conclure.
  • Ok, dans ce cas là on a $$

    \int_0^{2\pi} y'' \bar{y} dx= [y' \bar{y}]_0^{2\pi} - \int_0^{2\pi} y' (\bar{y})' dx= [y' \bar{y}]_0^{2\pi}- [y' \bar{y}]_0^{2\pi}=0.

    $$ Donc on conclut que $$

    \lambda \int_0^{2\pi} y \bar{y} dx =0.

    $$ Puisqu'on a supposé que $\lambda \neq 0$ et que par définition $y \bar{y} \geq 0$, on conclut que $y \bar{y}=0$ qui implique que $y=0$. Donc $y$ ne peut pas être une fonction propre si $\lambda = a+i b$ avec $b \neq 0$.
    Je pense que c'est bon maintenant.
    J'ai tout de même une question: pourquoi dans mon précédent post quand je remplace $y$ par $u+i v$ je n'obtiens pas le bon résultat ?
    Cordialement.
  • Bonjour,

    J’abandonne.
  • Bonjour
    on considère le problème
    $$
    \begin{cases}
    y''+\lambda y=0\\
    y(0)-2 y(2 \pi)=0\\
    y'(0)-y'(2\pi)=0
    \end{cases}
    $$
    je cherche à montrer que $\lambda = a+i b$ avec $b \neq 0$ ne peut pas être une valeur propre, c'est à dire que ce $\lambda$ donnera une solution trivial $y=0$.
    En suivant l'indication de bd2017, j'écris ceci:
    en multipliant les deux membre de l'équation par $\bar{y}$ et en intégrant sur $[0,2\pi]$ on obtient:
    $$
    \displaystyle\int_0^{2\pi} y'' \bar{y} dx + \lambda \displaystyle\int_0^{2\pi} y \bar{y} dx =0
    $$
    l'ipp nous donne que $\displaystyle\int_0^{2\pi} y'' \bar{y} dx= [y \bar{y}']_0^{2\pi}-\displaystyle\int_0^{2\pi} y' \bar{y}' dx= [y \bar{y}']_0^{2\pi} - [y \bar{y}]_0^{2\pi}$
    Donc on a
    $$
    [y \bar{y}']_0^{2\pi} - [y \bar{y}]_0^{2\pi}+ \lambda \displaystyle\int_0^{2\pi} y \bar{y} dx=0.
    $$
    J'ai des difficultés à conclure car le membre de droite de la dernière égalité n'est pas zéro. Comment faire?

    Cordialement
  • Personne?X:-(
  • Personne ne sait comment conclure? :-(
  • Bonjour,

    Tu as écrit $\displaystyle \int_0^{2\pi} y’(x) \bar{y}’(x ) dx=y(x)\bar{y}( x)|_0^{2\pi}.$

    C’est n’importe quoi, non ?
  • Bonjour
    c'est donc ça mon erreur, pardon c'était une énorme erreur d'inatention.
    On obtient ceci:
    $$
    y'(2\pi) \bar{y}(2\pi) -y'(0) \bar{y}(0) -\displaystyle\int_{0}^{2\pi} y' \bar{y}' dx + \lambda \displaystyle\int_0^{2\pi} y \bar{y} dx =0.
    $$
    En utilisant les conditions aux limites (donc $\bar{y}(0)= 2 y(2\pi)$ et $y'(0)= y'(2\pi)$ on a
    $$
    -y'(2\pi) \bar{y}(2\pi) -\displaystyle\int_0^{2\pi} \bar{y}' y' dx +\lambda \displaystyle\int_0^{2\pi} \bar{y} y dx =0
    $$
    J'ai du mal à conclure avec tout ça. Dans mon esprit il faut conclure que $y=0$, mais là je ne vois pas comment.
  • Bravo Yves m !
    Zorg69
  • Bonjour
    Bon je découvre que cela n'est pas encore terminé! Faudrait d'abord faire un calcul correct.
     
  • bd2017 mon calcul n'est pas encore correct? Où est-l'erreur? Car je crois avoir bien appliquer l'ipp
  • Et bien quand tu utilises les conditions de bords...
     
  • Bonjour,

    Enfin, ça a pris des plombes. Quand tu arrives à $\displaystyle -y'(2\pi) \bar{y}(2\pi) -\displaystyle\int_0^{2\pi} \bar{y}' y' dx +\lambda \displaystyle\int_0^{2\pi} \bar{y} y dx =0$ tu remaques que $\displaystyle \int_0^{2\pi} \bar{y} y dx =\int_{0}^{2 \pi} |y(x)|^2 dx \neq 0$ puisque $y \neq 0.$ Tu remarques aussi que $\displaystyle \int_0^{2\pi} \bar{y'(x)} y'(x) dx \in \R$ puisque cette quantité est égale à son conjugué complexe.

    Il ne manque plus qu'à demander à ceux qui pensent qu'on peut conclure de te montrer comment. :-D

    Cette approche ne marche que lorsque le terme intégré est réel, mais ce n'est pas le cas ici. Si c'était le cas, on aurait $a+\lambda b = 0$ avec $a, b$ réel et $b$ non nul : $\lambda$ est un réel.

    J'ai donné une démonstration.
  • Mais il y a toujours une erreur qui reste. Le premier terme qu'on obtient n'est pas $-y'(2\pi) \bar{y}(2\pi)$.
    Il faut remonter à l'origine de l'erreur. ( A priori $y$ n'est pas une fonction à valeurs réelles).
    En particulier on ne peut rien dire de $-y'(2\pi) \bar{y}(2\pi)$.
     
  • Bonjour,

    @mati : la méthode proposée par @bd2017 était une bonne idée, mais dans ce cas les conditions aux bords sont telles que le terme intégré n'est pas forcément réel : on ne peut pas conclure.

    Il te reste ma démonstration.
  • Merci beaucoup YvesM, donc on laisse tomber cette deuxième méthode car elle ne marche pas.
    Pour votre méthode je l'ai très bien compris.
  • Comment ça? Ma méthode marche très bien! @mati s'est trompé en remplaçant les conditions de bords.
    Une fois qu'il aura corrigé l'erreur, @Y.M tu verras qu'il n'y a pas de problème du tout.
     
  • Bonjour,

    Montre-nous. Je trouve comme @mati. Relis l'énoncé.
  • Ah pardon, à la fin c'est correct. Mais c'est parce qu'il a écrit.

    mati écrivait:
    > En utilisant les conditions aux limites (donc $\bar{y}(0)= 2 y(2\pi)$ et $y'(0)= y'(2\pi)$ on a ...

    A priori $y$ n'est pas réel. Donc la condition de bord étant $y(0)= 2 y(2\pi)$ on ne peut pas dire que $\bar{y}(0)= 2 y(2\pi).$

    Il s'est trompé à cet endroit c'est pour cela qu'ayant vu à la fin qu'il y a un problème, j'ai cru que son résultat final était faux.

    Mais alors ... avant d'en dire + je vérifie un truc et
     
  • Bonjour,

    Le terme intégré vaut $y'(b) \bar{y}(b) - y'(a) \bar{y}(a) $ avec $a=0$ et $b=2 \pi$. Les conditions aux bords sont $y(a) = 2 y(b)$ et $y'(a) = y'(b)$ donc on calcule $y'(b) \bar{y}(b) - y'(a) \bar{y}(a) = - y'(b) \bar{y}(b).$ Et on ne pas conclure.
  • Pour la méthode de YvesM:
    Les conditions aux limites nous donne le sysyème suivant:
    $$
    \begin{cases}
    (1-2e^{2\pi r}) C_1 +(1-2 e^{-2 \pi r}) C_2 =0\\
    (1-e^{2 \pi r}) C_1 +(e^{-2 \pi r}-1) C_2=0
    \end{cases}
    $$
    On calcul le déterminant de ce système qui nous donne
    $$
    det = 3 e^{-2 \pi r} + 3 e^{2 \pi r} -6
    $$
    Si $det \neq 0$ alors le système admet une solution unique $(C_1,C_2)=(0,0)$ par contre si $det =0$ alors le système admet une infinité de solutions et dans ce cas on pourra dire qu'il y a des solutions non trivial pour le problème est que $\lambda$ est une valeur propre.
    On remarque que $det=0$ si et seulement si $r=0$. donc pour le reste des valeurs de $r$, le determinant est toujours nul.

    Ma question est: je ne conclut pas de la même façon que YvesM car j'ai utilisé Cramer. Est-ce que ma conclusion est bonne?

    Cordialement
  • Bonjour,

    Tu ne sais pas résoudre $e^z=1$ pour $z\in \C.$
  • pourquoi on va résoudre l'équation $e^z=1$? A quoi correspond cette équation?
  • supp
  • Bonjour,

    @side : la condition au bord $y(0)=2 y(2 \pi)$ empêche la fonction $y$ d’être $2\pi$-périodique.

    Les solutions sont toutes triviales.

    C’est la particularité de ce système : il n’a pas d’autres solutions que triviales.

    @mati : Tu as écrit que le déterminant est nul si et seulement si $r=0$ : c’est très faux. Relis mon message. A quoi ça sert que Ducros se décarcasse ?
  • Oui mais vous avez divisé sur 2 et je ne vois pas pourquoi. Moi je dis que $$

    e^{2 \pi r}=1 \iff 2 \pi r = 2 i \pi k, \ k \in \mathbb{Z} \iff r=i k, \ k \in \mathbb{Z}.

    $$ Je me trompe ?
  • Oui c'est ça. Les valeurs propres sont $k^2$ et les fonctions propres $y(x)=\sin(kx).$
     
  • Merci. Donc il y a une erreur de frappe dans le résultat donné par YvesM. On ne divise pas par 2.
  • Rebonjour
    Le calcul explicite proposé par @Y.M n'est pas vraiment difficile.
    Pour revenir à ma méthode, ce qui a posé problème c'est la "non symétrie" de la condition $y(0)=2y(2\pi).$
    Normalement si les valeurs propres sont réelles les termes de bords après intégration par parties doivent disparaître.
    Et effectivement elles sont réelles. Alors il y avait à faire un travail en plus.

    Pour cela on multiplie la première équation par y' et on intègre pour avoir
    $ y'^2+ \lambda y^2= cste.$ En particulier $y'^2(0)+ \lambda y^2 (0) =y'^2(2\pi)+ \lambda y^2 (2\pi) $

    Avec les conditions de bord, cette dernière égalités conduit à $y(0)=y(2\pi)=0.$

    Si on reprend ma méthode on obtient alors

    $ \int _0^{2\pi} |y'|^2 + \lambda \int _0^{2\pi} |y|^2=0$ ce qui implique que $\lambda$ est réel.
     
  • supp
  • @side La question c'est pas de prouver que y est nul. Il s'agit de montrer que trouver les valeurs propres de l'opérateur -Laplacien (+ BC) sur $[0, 2\pi]$ sont réelles.

    Soit on montre que c'est bien le cas, comme finalement j'ai fait.
    Soit on fait un calcul explicite comme proposé par Y.Meyer et c'est fait par @mati. J'ai résumé en donne les v.p et fonctions propres

    Remarque. Le calcul explicite donne les mêmes valeurs propres et fonction propres que l'opérateur -Laplacien avec condition de Dirichlet (i.e $y(0)=y(2\pi)$ ). Ce que je trouve étonnant.
     
  • supp
  • supp
  • Bonjour,

    Cet exercice est tordu parce qu’on donne des conditions aux bords et qu’en fait il faut les préciser pour terminer le calcul. Par exemple on donne $y(0)=2 y(2\pi)$ mais en fait on a nécessairement $y(0)=2y(2 \pi)=0$ ce qu’on doit d’abord prouver.

    C’est ma réponse à la question de @d2017 : on pourrait donner $y(0)=k y(2 \pi)$ pour tout $k$ on obtiendrait les mêmes solutions...

    Ma méthode (une parmi d’autres) permet de répondre à la question posée sans démontrer d’autres caractéristiques des solutions.

    @mati :
    Tu dois écrire ce que tu cherches à démontrer. Écrire une démonstration digne de ce nom, puis demander de l’aide. Mais si tu poses des questions vagues on ne sait pas comment t’aider. J’ai donné la démonstration pour $r$. Essaie de la refaire. Si tu veux faire autre chose, libre à toi, mais expose ta problématique clairement.
  • YvesM écrivait a écrit:
    Cet exercice est tordu parce qu’on donne des
    conditions aux bords et qu’en fait il faut les
    préciser pour terminer le calcul.

    Bonjour Cher YvesM, je crois que tu as oublié qu'on a déjà traiter cet exercice avec remarque au moins deux fois . Par exemple dans le fil de bib et tu as donné une belle preuve ( la même qu'ici) http://www.les-mathematiques.net/phorum/read.php?4,1462356,1462464#msg-1462464

    Dans ce fil, j'ai eu aussi l'idée comme bd de multiplier par $y'$ et j'ai trouvé aussi les conditions $y(0)=y(2\pi)=0$ mais ma étourderie à l’époque m'a empêché de l'exploiter http://www.les-mathematiques.net/phorum/read.php?4,1462356,1463052#msg-1463052
    @bd2017
    Sans ce message http://www.les-mathematiques.net/phorum/read.php?4,2011754,2017984#msg-2017984
    Ton $k$ c'est un entier ou un réel ?
    @mati oui il y a une coquille dans la preuve de YvesM ( division par 1/2)
    Le 😄 Farceur


Connectez-vous ou Inscrivez-vous pour répondre.